You are on page 1of 37

0484-3190310

9446331522
9446331522
TARGET PRELIMS 2017
100 IMPORTANT POLITY QUESTIONS FROM NEO IAS MODEL TESTS
1. With reference to Presidents rule in India, which of the following statements is correct?
(a) President can proclaim failure of constitutional machinery in a State on the basis of report
of the Governor only.
(b) Imposition of Presidents Rule is totally immune from judicial review.
(c) The President cannot take over to himself the powers that are vested in the High Court of
the State.
(d) The President can extend the duration of Proclamation beyond normal one year period on
the basis of report of the Governor.

Answer (c)
Explanation
Option (c) is correct. When Presidents Rule is imposed, the President may take over the
executive and legislative functions. But the President does not have the power to assume the
functions of the High Court. All other statements are wrong. The President may issue
proclamation of emergency on the basis of report of Governor of a State or otherwise.
Presidents Rule can be subject to judicial review. The judiciary can check whether Article
356 has been misused or not. If it is to be extended beyond normal one year period, two
conditions are to be satisfied firstly, national emergency in force whole or part of the state;
secondly, certification by the Election Commission that under prevailing conditions general
election to the Assembly of the State cannot be held. It is not on the basis of the report of the
Governor.

2. Consider the following statements:


1. A proclamation issued under Article 356 must be approved by resolutions of both Houses
of Parliament within a period of one month from the date of proclamation.
2. When Article 356 is imposed, the President can by proclamation assume to himself the
powers and functions of the State Government.
3. If the Houses of Parliament approves the proclamation, it shall be in force for a period of
six months.
Which of the statements given above is/are correct?
(a) 1, 2 and 3 (b) 1 and 2 only
(c) 2 and 3 only (d) 3 only
Answer: (c)
Explanation
1st statement is wrong. The Proclamation under Article 356 must be laid before both Houses
of Parliament within two months from the date of proclamation. It is within two months and
not within one month. Other statements are correct. The President can assume to himself
the functions of the State Government when Presidents Rule is imposed in a State. The
Proclamation will be in force for six months if it is approved by both Houses of Parliament.

3. Consider the following statements:


1. Article 19(2) provides defamation as a restriction to freedom of speech and expression
guaranteed under Article 19(1)(a) of the Constitution.
2. Right to reputation is a part of right to life guaranteed under Article 21 of the Constitution.
Select the correct answer using the code given below.
(a) 1 only (b) 2 only
(c) Both 1 and 2 (d) Neither 1 nor 2

Answer: (c)
Explanation
Both statements are correct. Article 19(1) (a) gives to each and every citizen right to freedom

of speech and expression. Article 19(2) provides restrictions to the freedom of speech and
expression. Defamation is one of the restrictions. The Supreme Court in the case of
Subramanian Swamy v. Union of India recognised right to reputation as a right under right
to life guaranteed under Article 21 of the Constitution.

4. Which of the following statements is/are correct regarding Governor?


1. The Governor with the aid and advice of the Chief Minister and Council of Ministers sends
report to the President about the failure of the constitutional machinery in the State.
2. Article 163 provides that Governor has the power to decide the matters which falls within
his discretion.
Select the correct answer using the code given below.
(a) 1 only (b) 2 only
(c) Both 1 and 2 (d) Neither 1 nor 2
Answer: (b)
Explanation
1st statement is not correct. The Governor has two types of powers and functions firstly,
functions exercised with the aid and advice of the Chief Minister and Council of Ministers;
secondly, discretionary powers. Discretionary powers are exercised by the Governor with his
own individual judgment or without the aid and advice of the Chief Minister and Council of
Ministers. Sending report to the President about the failure of the constitutional machinery in
a State is a discretionary power exercised by the Governor. It is not exercised in accordance
with the aid and advice of the Chief Minister and the Council of Ministers. Second statement
is correct. Article 163 deals with discretionary powers of the Governor. The Constitution
provides that the Governor himself can decide the matters in which he can exercise
discretionary powers.

5. Consider the following statements:


1. Article 47 of the Indian Constitution provides that the State shall endeavour to bring about
prohibition of the consumption except for medicinal purposes of intoxicating drinks and of
drugs which are injurious to health.
2. Article 47 is justiciable in nature.
Which of the statements given above is/are correct?
(a) 1 only (b) 2 only
(c) Both 1 and 2 (d) Neither 1 nor 2
Answer: (a)
Explanation
First statement is correct. Article 47 provides that State shall prohibit consumption of
intoxicating drinks and drugs which are injurious to health. But medicinal purposes are
exempted. Second statement is wrong. Article 47 is a Directive Principle of State Policy. These
are directives to the State and cannot be enforced in courts and hence are not justiciable in
nature.

6. In the Seventh Schedule of the Indian Constitution, which of the following entries is/are
included in the Concurrent List?
1. Labour
2. Land
3. Education
4. Forests
5. Agriculture
Select the correct answer using the code given below.
(a) 3 only (b) 1, 2 and 5
(c) 1 and 4 (d) 1, 3 and 4

Answer: (d)
Explanation
Seventh Schedule of the Constitution divides the legislative power of the Union and States.
There are three Lists in the Seventh Schedule. Union List enumerates the subjects upon
which the Parliament can make laws. State List specifies subjects upon which the State


Legislature can make laws. Concurrent List provides subjects upon which both the
Parliament and State Legislature can make laws. Labour, Forests and Education are subjects
included in the Concurrent List. Land and Agriculture are entries mentioned in the State
List.

7. With reference to the original jurisdiction of the Supreme Court, which of the following
statements is/are correct?
1. The Supreme Courts original jurisdiction extends to any dispute between the
Government of India and one or more States.
2. The Supreme Courts jurisdiction in matters which may specifically be referred to it by the
President of India under Article 143 of the Constitution is known as original jurisdiction.
Select the correct answer using the code given below.
(a) 1 only (b) 2 only
(c) Both 1 and 2 (d) Neither 1 nor 2
Answer: (a)
Explanation
First statement is correct. The Supreme Court exercises mainly four types of jurisdiction
original jurisdiction, writ jurisdiction, appellate jurisdiction, advisory jurisdiction and revisory
jurisdiction. Original jurisdiction is exercised to settle disputes between Centre and States
and between States. Second statement is wrong. If the Supreme Court expresses an opinion
on a matter referred by the President, it is advisory jurisdiction. It does not come within the
purview of original jurisdiction.

8. Consider the following statements:


1. Article 44 provides that the State shall endeavour to secure for the citizens a uniform civil
code throughout the territory of India.
2. The Government of India enacted the Muslim Women (Protection of Rights on Divorce)
Act, 1986 in order uphold the Shah Bano case (1985) judgement pronounced by the
Supreme Court of India.
Which of the statements given above is/are correct?
(a) 1 only (b) 2 only
(c) Both 1 and 2 (d) Neither 1 nor 2

Answer: (a)
Explanation
First statement is correct. Article 44 is a Directive Principle of State Policy. It directs the State
to secure uniform civil code for its citizens. Presently, there are different laws governing
personal matters for different communities in India. A uniform civil code means unifying all
the personal laws to have one set of secular laws dealing with these aspects that will apply to
all citizens of India irrespective of their community. Second statement is wrong. The Supreme
Court in Shah Bano case held that a Muslim woman is entitled to maintenance like any other
Indian woman. The government enacted the Muslim Women (Protection of Rights on Divorce)
Act, 1986, to nullify the Shah Bano verdict. Though the proclaimed objective of the law is
to protect the rights of Muslim women who have been divorced by, or have obtained divorce
from, their husbands.
9. Consider the following statements:
1. The protection and improvement of lakes and rivers is a fundamental duty enshrined in
the Constitution.
2. As per Indian Constitution, the State Government doesn't have power to make laws for
any of the water resources.
3. The power to legislate the regulation and development of interstate rivers lies with the
Parliament.
4. The legislative framework of the Constitution relates to water is based on Entry 17 of the
State List, Entry 56 in the Union List, and Article 262 of the Constitution.
Which of the statements given above is/are correct?
(a) 1 and 2 only (b) 3 and 4 only
(c) 1, 3 and 4 only (d) 1, 2, 3 and 4


Answer (c)
Explanation
Water is one of the most important resources for any country and the majority of this water is
obtained from rivers to meet the needs of irrigation, cattle rearing and various other
sanitation purposes by our states. These rivers, flowing in all directions passing through
different states in India are either intra-state (flowing within a single state from the point of
its source to its mouth) or inter-state river (which flows through the boundary of two or more
states), which, in some cases have even led to disputes among the states. And this is where
the role and the power of Union government comes into play, i.e. to manage these inter-state
rivers and tackle any related disputes. It is also enshrined in our constitution that it is our
fundamental duty to protect and improve the natural environment including forests,
lakes, rivers, and wild-life and to have compassion for living creatures.
As per Indian constitution, a state government has the power to make laws for the water
resources of that state. Under Entry 17 of the state list, the legislative power of a state has
to be exercised without adversely affecting the interests of other states and avoiding any
dispute. But since the power to legislate the regulation and development of interstate
rivers lies with the Parliament, the authority of the state Government over water can be
exercised, but it will be subjected to limitations that can be imposed by the Parliament.
Thus, it wont be right if we say that water is entirely a state-subject. Rather, it is as much a
Union subject as it is a state subject as the supremacy in all its matters lies with the
Parliament. The legislative framework of the constitution related to water is based on
Entry 17 of the State List, Entry 56 in the Union List, and Article 262 of the Constitution.
Reference: Yojana July 2016, page 19

10. In the context of river water disputes in India, consider the following statements:
1. Presently, there are separate tribunals set up for adjudicating each inter-state river water
disputes.
2. The Centre has recently decided to set up a single, permanent tribunal to adjudicate all
inter-state river water disputes, with a retired Supreme Court judge as its Chairperson.
3. The Centre has also proposed to float some Benches to look into such disputes and these
will cease to exist once the disputes are resolved.
4. Inter-state Water Disputes Act, 1956 will be amended to set up Dispute Resolution
Committee to handle disputes prior to the Tribunal.
Which of the statements given above is/are correct?
(a) 1, 3 and 4 only (b) 2 and 3 only
(c) 1, 2 and 4 only (d) 1, 2, 3 and 4
Answer (d)
Explanation
All the statements are correct.

11. What is/are the major difference/differences between a written and an unwritten
constitution?
1. An unwritten constitution is the formal source of all constitutional laws in the country
and the written constitution is not the formal source.
2. In written constitution, conventions play a dominant role while they do not have any role
in unwritten constitution.
Select the correct answer using the code given below.
(a) 1 only (b) 2 only
(c) Both 1 and 2 (d) Neither 1 nor 2

Answer (d)
Explanation
If the provisions of the constitution are codified into a single legal document, it is a written
constitution whereas unwritten constitution means its provisions are not codified into a
single legal document. Since its provisions are codified in a written form, it serves as the
basic law of the land and all other laws are originated from it. Hence, written constitution is
formal source of all laws including constitutional law. On the other hand, unwritten
constitution is evolved and developed over period of time and conventions, customs and laws
play a dominant role in the governance of the country.


12. With reference to Constitutional Government, which of the following statements is/are
correct?
1. It places effective restrictions on individual liberty in the interest of State authority.
2. It places effective restrictions on the authority of the State in the interest of individual
liberty.
3. The powers and authority of government is defined, distributed and limited by the
Constitution.
4. The Constitution is superior to all ordinary Acts of Legislatures.
Select the correct answer using the code given below.
(a) 1, 2 and 3 only (b) 1, 2 and 4 only
(c) 2, 3 and 4 only (d) 1, 2, 3 and 4

Answer (c)
Explanation
Constitutional government is defined by the existence of a constitution in which the powers
and authority of government is defined, distributed and limited by the fundamental law - the
Constitution. It places effective restrictions on the authority of the State in the interest of
individual liberty. (Hence statement 2 is correct and statement 1 is wrong). In a constitutional
Government the constitution is superior to all ordinary Acts of Legislatures.

13. With reference to the Preamble, which of the following statements is/are not correct?
1. Preamble is not a part of the Constitution.
2. The Preamble has been amended only once so far.
3. It is non-justiciable in nature.
Select the correct answer using the code given below.
(a) 1 only (b) 1 and 2 only
(c)2 and 3 only (d) 1 and 3 only

Answer (a)
Explanation
In the Kesavananda Bharati v. State of Kerala (1973) case, The Supreme Court reversed an
earlier verdict and held that the Preamble formed part of the Constitution because it was
separately passed after the enacting provisions had been passed. Hence Statement 1 is not
correct. The Preamble has been amended only once so far, in 1976, by 42nd Constitution
Amendment Act, which has added three new words - Socialist, Secular and integrity to the
Preamble. The Preamble is non-justiciable i.e. its provisions are not enforceable in a court of
law.

14. With reference to Bills under Article 3, consider the following statements:
1. No Bill can be introduced in either House of Parliament except on the recommendation of
the President.
2. No Bill can be introduced in either House of Parliament before referring it to the
Legislatures of the concerned States for expressing opinion thereon.
3. The views expressed by the State Legislature on the Bill are binding on the President.
Select the correct answer using the code given below.
(a) 1 only (b) 1 and 2 only
(c) 2 and 3 only (d) 1 and 3 only

Answer (b)
Explanation
Under Article 3, the Parliament is empowered:
1. to form a new State by separating a part of the territory of a State or by uniting two or
more States or parts of States or by uniting any territory to a part of any State.
It also includes the power of the Parliament to form a new State or Union territory by
uniting a part of any State or Union territory to any other State or Union territory.
2. to increase the area of any State
3. to diminish the area of any State
4. to alter the boundaries of any State
5. to alter the name of any State


The term State includes a Union territory in all the above five cases. However, a Bill under
Article 3 must satisfy two conditions:
1. No Bill for the purpose can be introduced in either House of Parliament except on the
recommendation of the President.
2. The Bill must be referred by the President to the Legislatures of the concerned States for
expressing opinion thereon. The President should do so before giving his
recommendation. Hence no Bill can be introduced in either House of Parliament before
referring it to the Legislatures of the concerned States for expressing opinion thereon.
Such an opinion has to be expressed within a period specified by the President. In any
case, the views expressed by the State Legislature are not binding on the President
or Parliament.

15. Regarding the Eighth Schedule of the Constitution, consider the following statements:
1. Rajasthani and Bhojpuri are included in this schedule.
2. The provisions in this schedule can be amended by simple majority
3. Presently there are only 22 languages in this schedule.
Select the correct answer using the code given below.
(a) 1 only (b) 2 only
(c) 3 only (d) None

Answer (c)
Explanation
Eighth Schedule includes languages that are recognized by the Constitution. Recently, there
have been demands to include Rajastani and Bhojpuri languages that are spoken by a large
number of people in this Schedule. But they have not been included so far. The provisions in
this schedule can be amended under Article 368 only. They cannot be amended by simple
majority. As of now 22 languages have been included in the Eighth Schedule.

16. Consider the following:


1. Freedom of speech and expression
2. Right to equal access to public employment
3. Protection against untouchability
Which of the above fundamental rights is/are available to aliens?
(a) 1 only (b) 1 and 2 only
(c) 3 only (d) 1, 2 and 3

Answer (c)
Explanation
FUNDAMENTAL RIGHTS AVAILABLE ONLY TO CITIZENS
1. Prohibition of discrimination on grounds of religion, race, caste, sex or place of birth
(Article 15).
2. Equality of opportunity in matters of public employment (Article 16).
3. Protection of six fundamental freedoms - speech and expression, assembly, association,
movement, residence and profession (Article 19).
4. Protection of language, script and culture of minorities (Article 29).
5. Right of minorities to establish and administer educational institutions. (Article 30).

17. With reference to the Ninth Schedule of the Constitution, consider the following statements:
1. It was added to the constitution by the First Amendment Act 1951.
2. All the provisions in the Ninth Schedule are open to judicial scrutiny on the ground of
violation of Fundamental rights.
Select the correct answer by choosing the code given below.
(a) 1 only (b) 2 only
(c) Both 1 and 2 (d) Neither 1 nor 2

Answer (a)
Explanation
Ninth Schedule was added to the constitution by the First Amendment Act 1951. Provisions
added to 9th schedule after April 24, 1973 is open to judicial review on the ground of violation
of Fundamental rights.


18. With reference to the basic structure doctrine, consider the following statements:
1. The Constitution of India does not explain the basic structure of the Constitution.
2. The basic structure doctrine in India is a legislative innovation to check executive
overreach.
3. Every provision of the Constitution can be amended provided the basic structure of the
Constitution remains the same.
Select the correct answer using the code given below.
(a) 1 only (b) 3 only
(c) 1 and 3 only (d) 1, 2 and 3

Answer (c)
Explanation
The Constitution of India does not explain the basic structure of the Constitution. However,
the Supreme Court in Kesavananda Bharatis case held that every provision of the
Constitution can be amended provided the basic structure of the Constitution remains the
same. Hence, the basic structure doctrine is a judicial innovation in India. The basic features
cannot be altered in exercise of the power under Article 368.

19. Which of the following statements is/are correct?


1. Certiorari is a writ which is issued by a superior court against the orders of an inferior
court or tribunal.
2. Writ of prohibition is issued while deciding the legality of an ongoing proceeding before a
Lower Court/Tribunal/Authority.
3. Both writs are supervisory in nature and the Writ Court can decide the case by considering
all the issues raised by the parties.
4. The object of both writs is to secure that the jurisdiction of an inferior court or tribunal is
properly exercised and that it does not usurp the jurisdiction which it does not possess.
Select the correct answer using the code given below.
(a) 1 and 2 only (b) 3 and 4only
(c) 1, 2 and 3 only (d) 1, 2, 3 and 4

Answer (d)
Explanation
Certiorari is a writ which is issued by a superior court against the orders of an inferior
court or tribunal. It is issued if an order has been obtained by fraud or collusion or there is
an error in the order even though acted within the jurisdiction. The writ of prohibition is
a writ issued by the Supreme Court or a High Court forbidding the inferior court to
continue proceedings therein in excess of its jurisdiction or to usurp a jurisdiction with
which it is not legally vested. Writ of prohibition is issued while deciding the legality of an
ongoing proceeding before a Lower Court/tribunal/Authority. The object of both writs is to
secure that the jurisdiction of an inferior court or tribunal is properly exercised and that it
does not usurp the jurisdiction which it does not possess

20. With reference to loss of citizenship in India, consider the following statements:
1. Deprivation is a voluntary act by which a person holding the citizenship of India, who is
also a national of another country, may abjure it.
2. Renunciation is a compulsory termination of the citizenship of India, by an order of the
Government of India.
Select the correct answer using the code given below.
(a) 1 only (b) 2 only
(c) Both 1 and 2 (d) Neither 1 nor 2

Answer (d)
Explanation
Renunciation is a voluntary act by which a person holding the citizenship of India, who is
also a national of another country, may abjure it. A person who is a major and has legal
capacity (sound mind etc.) can renounce Indian citizenship by making a declaration to that
effect. Deprivation is a compulsory termination of the citizenship of India, by an order of the
Government of India. Deprivation is applicable to a person who is a citizen of India by
naturalization or by virtue of close (c) of Article 5 or by registration.


21. Which one of the following statements is not correct?
(a) The power of the Governor to grant pardon is equal to the power of the President
(b) The power of High Courts to issue writs under Article 226 of the Constitution of India is
wider than the power of the Supreme Court under Article 32
(c) The Governor has no power to appoint Judges of the State High Court.
(d) The District Judge is the highest judicial authority in the district in both civil and criminal
matters.

Answer (a)
Explanation
The power to grant pardon by the president is wider than that of the governor. President can
grant pardon to a death convict (Article 72(1) (c)). He can also pardon punishment inflicted
by the court martial (Article 73(1) (a)). Governor do not have these powers.
High Courts have powers to issue writs under Article-226. High courts can enforce
Fundamental Rights (FR) or any other legal rights through writs. But under article 32,
Supreme Courts power to issue writs is confined to matters regarding fundamental rights
only.
The Governor has no power to appoint Judges of the State High Court but he is entitled to be
consulted by the President in the matter.
District judge is the highest judicial authority in the District (civil and criminal).

22. Which of the following is/are correct regarding the office of Governor in India?
1. One Governor can act as Governor of more than one State
2. The Governor has the power to grant pardons, reprieves, remission of punishment to
persons convicted under the State Law
3. The Governor is appointed by the President on the recommendation of the Chief Minister
of the respective State
Select the correct answer using the code given below.
(a) 1 and 2 only (b) 1, 2 and 3
(c) 2 and 3 only (d) 1 only

Answer: (a)
Explanation
President appoints Governor on the recommendations of the Central Government.
Normally there shall be a governor for each state. But as per an amendment made in 1956, it
is possible to appoint the same person as the governor for the two or more states (Article
153).
The governor has the power to grant pardons, reprieves, respites or remission of punishment
or commute the sentence (Article 161).
23. Which of the following statements is/are correct?
Under Article 200 of the constitution of India, the Governor of a State may
1. Withhold his assent to a bill passed by the State Legislature.
2. Reserve the bill passed by the State Legislature for consideration of the President.
3. Return the bill, other than a money bill, for reconsideration of the Legislature.
Select the correct answer using the code given below.
(a) 1 only (b) 1 and 2 only
(c) 2 and 3 only (d) 1, 2 and 3

Answer: (d)
Explanation
When a bill is sent to the governor after it is passed by state legislature, he can declare either
that he assents to the Bill or that he withholds assent or that he reserves the Bill for the
consideration of the President. He can also return the bill, other than a money bill, for
reconsideration of the president. Article 200 also add that it is obligatory upon the governor
to send the bill for the consideration of the president if it endanger the position of state high
courts.

24. With regard to the administration of Scheduled Areas under Article 244 of the Constitution of
India, which one of the following statement is not correct?

(a) These are Areas inhibited by Tribes specified as Scheduled Tribes in States other than
Assam, Mizoram Tripura and Meghalaya.
(b) The Scheduled Areas are administered by an Advisory Council.
(c) The Governor of the State may direct that a particular Act of the Parliament shall not
apply to a Scheduled area.
(d) The Governor can increase the area of any Scheduled Area.

Answer: (d)
Explanation
Altering, increasing, decreasing, incorporating new areas, or rescinding any orders relating to
Scheduled Areas can be done only by presidential order which is issued in consultation
with the respective state government.

25. Consider the following statements about Panchayati Raj Institution in India:
1. Courts shall have no jurisdiction to examine the validity of a law, relating to delimitation
of constituencies or the allotments of seats, made under Article 243K.
2. Every five years the Governor of a State shall appoint a Finance Commission to review the
financial position of the Panchayats and to make recommendations.
3. The report of the State Finance Commission, together with a memorandum of action
taken on it, shall be laid before the Parliament.
Which of the statements given above is/are correct?
(a) 1, 2 and 3 (b) 1 and 2 only
(c) 2 and 3 only (d) 1 only

Answer: (b)
Explanation
Under Article 329, Courts have no jurisdiction to examine the validity of a law relating to
delimitation of constituencies or the allotment of seats made under Article 243K. An
election to a Panchayat can be called in question only by an election petition which should be
dealt in the manner prescribed by the state law on the matter. Article 243 I of the Indian
Constitution prescribes that in every five years the Governor of a State shall appoint a
Finance Commission to review the financial position of the panchayats and to make
recommendations. The report of the State Finance Commission and the memorandum of
action taken shall be laid before state legislature, not before the Parliament.
26. The Eleventh Schedule inserted by the 73rd Amendment Act distributes powers between the
State Legislatures and the Panchayats, and lists 29 items. Which among the following are
listed in the Eleventh Schedule?
1. Minor irrigation
2. Animal husbandry
3. Women and child development
4. Fire service
5. Cattle ponds, prevention of cruelty to animals
Select the correct answer using the code given below.
(a) 1, 2 and 3 only (b) 1, 4 and 5 only
(c) 2, 3 and 4 only (d) 3, 4 and 5 only

Answer: (a)
Explanation
Fire Service, cattle ponds and prevention of cruelty to animals is not included in the eleventh
schedule. Prevention of cruelty to animals is in the concurrent list.
27. The 73rd Amendment of the Constitution provided Constitutional status to the Panchayati Raj
Institutions. Which of the following are the main features of this provision?
1. A three tier system of Panchayati Raj for all States.
2. Panchayat elections in every 5 years.
3. Not less than 33% of seats are reserved for women.
4. Constitution of District Planning Committees to prepare development plans.
Select the current answer using the code given below.
(a) 1, 2 and 3 only (b) 1, 3 and 4 only
(c) 1, 2 and 4 only (d) 2, 3 and 4 only

Answer: (a)
Explanation
Constitution of District Planning Committees and Metropolitan Committees are provided in
74th Amendment which deals with Municipalities. Provisions for regular panchayat
elections, constitution of State finance and Elections Commissions, reservations are
incorporated in both 73rd and 74th amendment act. Whereas constitution of ward
committees and institutions for planning are provided only in 74th amendment act.

28. Which one of the following statements is not correct?


(a) The Council of States consists of elected and nominated members
(b) The Council of states is not subject to dissolution
(c) One-third of the members of the Council of States retire every five years
(d) The Vice President of India is the ex-officio Chairman of the Council of States
Answer: (c)
Explanation
The council of states or Rajya Sabha is not subject to dissolution. It is a permanent body, but
1/3 of its members retire on the expiration of every second year, after the completion of their
6 year tenure. Rajya Sabha also contains 12 nominated members. They are nominated by the
President from amongst persons having special knowledge or practical experience in
literature, science, art and social service. The Vice President becomes the ex-officio
Chairman of the House.

29. The Committee on Public Accounts consists of


(a) 15 members from the Lok Sabha and 7 members from the Rajya Sabha
(b) 20 members from the Lok Sabha and 20 members from the Rajya Sabha
(c) 20 members from the Lok Sabha and 10 members from the Rajya Sabha
(d) 30 members from the Lok Sabha and 15 members from the Rajya Sabha

Answer: (a)
Explanation
Public Account Committee is one of the most important committees of the Parliament that
helps in having effective financial control over executive. Estimates Committee and
Committee on Public Undertaking are other committees for financial control. Some key
facts about the committee are:
Members of these committees are elected
Chairpersons are appointed by the Speaker.
No minister can be member of these committees.
The term of the office of the members is one year
Committee Purpose Members Chairman
Public Accounts Examine audit report of the 22(15 from Lok Sabha From its members
Committee CAG and 7 from Rajya Sabha)

Estimates Examine the estimates of 30 All from Lok Sabha From its members
Committee Budgets and suggest
improvements
Committee on Examine reports and accounts 22(15 from Lok Sabha From its Lok
Public Undertaking of Public Undertaking and 7 from Rajya Sabha) Sabha members
only

30. Which of the following statements is not correct?


(a) The Parliament cannot enact legislation on water
(b) The Constitution of India does not contain express provisions regarding secret session,
but the parliament can have secret session
(c) The Prime Minister must disclose to the President such information pertaining to the
decisions of the Council of Ministers relating to the administration of the affairs of the
union.
(d) The Parliament also includes the President of India


Answer: (a)
Explanation
Even though water is incorporated in the state list of the seventh schedule, union has the
authority to legislate on regulation and development of inter-State rivers and river valleys.
Therefore, parliament can legislate on matters pertaining to inter-state river, water-sharing
etc.
The President has the right of receiving or calling for information. Under article 78 of the
Constitution, the Prime Minister has to communicate to the President all decisions of the
Council of Ministers relating to the administration.
Parliament doesnt contain express provisions on secret sitting. Under article 361,
constitution just provides protection of publication of proceedings in the secret sitting. But
detailed description of secret sitting is found in the Rules of Procedure of the parliament.
Indian Parliament consists of Lok Sabha, Rajya Sabha and the president.

31. Consider the following statements about the Estimates Committee:


1. Members of the Committee are drawn from both the Lok Sabha and the Rajya Sabha.
2. The maximum number of members of the Committee is 30.
3. The members are nominated by the Speaker and the Chairman.
Which of the statements given above is/are correct?
(a) 1, 2 and 3 (b) 2 and 3 only
(c) 2 only (d) 1 and 3 only

Answer: (c)
Explanation
Estimates Committee consists of 30 members who are elected by the Lok Sabha every year
from amongst its members.

32. Which one among the following statements about the Legislatures of India is not correct?
(a) The President may prorogue the Houses or either House of Parliament and dissolve the
Lok Sabha.
(b) In the case of the State Legislatures, it is the Governor who may from time to time
prorogue the Houses or either House of the Legislature and dissolve the Legislative
Assembly.
(c) The proposal to summon, prorogue or dissolve the Lok Sabha may be made by the Prime
Minister with or without the consultation of the Cabinet.
(d) In no case should the time period intervening between the last sitting in one session of
Parliament and the date appointed for its first sitting in the next session exceed thirty-six
weeks.

Answer: (d)
Explanation
Article 85(1) states that The President shall from time to time summon each House of
Parliament to meet at such time and place as he thinks fit, but six months shall not
intervene between its last sitting in one session and the date appointed for its first sitting in
the next session.
The power to summon, prorogue or dissolve the Lok Sabha is vested in the President. He
exercises this power on the recommendation of the Prime Minister or the Cabinet.
Similar is the case with the governor as well.

33. Which of the following are the reasons behind the Constitutional design of Rajya Sabha?
1. Mature input in legislative and constituent process from more experienced persons.
2. Tending some degree of continuity to the parliamentary work in view of the fact that the
Rajya Sabha is a continuous chamber.
3. Unequal weightages to votes in Rajya Sabha since there is equal number of seats given to
all states.
Select the correct answer using the code given below.
(a) 1, 2 and 3 only (b) 1 and 2 only
(c) 2 and 3 only (d) 1 and 3 only


Answer: (b)
Explanation
Rajya Sabha enables representation to all sections of the society and to all parts of the
country. Besides, the participation of members of Rajya Sabha in legislative discussion can
provide mature and valuable inputs. A bicameral legislature has some more advantages. A
bicameral legislature makes it possible to have every decision reconsidered. Being a
permanent house, it also provide some degree of continuity to the parliamentary work.
However, representation of states in Rajya Sabha is asymmetrical. It means that each states
representation in the house varies widely with the population. Where Uttar Pradesh has 31
MPs in Rajya Sabha, north eastern states send only one representative each.
34. Which of the following statements related to Money Bills is not correct?
(a) It cannot be introduced in the Council of States
(b) If any question arises whether the Bill is Money Bill or not, the decision of the Speaker is
final
(c) In case of deadlock over a Money bill, the President can summon a joint sitting of the
Parliament
(d) A Money Bill cannot be introduced expect on the recommendation of the President

Answer: (c)
Explanation
There is no scope to arise a deadlock in the case of money bill. Because Rajya Sabha has no
role in amending the money bill. It can only suggest recommendations subject to the approval
of Lok Sabha. Therefore, joint sitting need not be or cannot be summoned in passing money
bill.
A money bill is introduced in the Lok Sabha only with the prior recommendation of the
president. The authority to decide whether a bill is money bill or not rest with the speaker.
And his decision on the matter is final.

35. Which of the following expenditure is/are charged on the Consolidated Fund of India?
1. The emoluments and allowances of the President and the expenditure relating to his/her
office.
2. The salaries and allowances of the Chairman and Deputy Chairman of the Council of
States and the Speaker and Deputy Speaker of the House of the People.
3. Debt charges for which the Government of India is liable.
Select the correct answer using the code given below.
(a) 1 and 2 only (b) 1, 2 and 3
(c) 2 and 3 only (d) 3 only

Answer: (b)
Explanation
Following are the charged expenditures: Salary and Allowances of the President, Speaker /
Deputy speaker of Lok Sabha, Chairman/ Deputy chairman of Rajya Sabha, Salaries and
Allowances of Supreme Court judges, Pensions of Supreme Court as well as High Court
Judges, Salaries and Allowances of CAG, Lok Pal and Debt charges of Government of India.
The above expenditures cannot be voted because these payments are deemed to be
guaranteed by the state.

36. A person shall be disqualified for being chosen as, and for being, a member of either House of
the Parliament if she/he

1. Holds any office of profit under the Government of India or the Government of any State,
other than an office exempted by the Parliament by law
2. Is of unsound mind and stands so declared by a competent court
3. Remains absent from all meetings of the House for a period of 60 days without the
permission of the House
Select the correct answer using the code given below.
(a) 1 and 2 only (b) 2 and 3 only
(c) 1, 2 and 3 (d) 1 and 3 only


Answer: (c)
Explanation
According to Article 102.(1) A person shall be disqualified for being chosen as, and for being,
a member of either House of Parliament
o if he holds any office of profit under the Government of India or the Government of any
State, other than an office exempted by the parliament by law.
o if he is of unsound mind and stands so declared by a competent court;
o if he is an undischarged insolvent;
o if he is not a citizen of India, or has voluntarily acquired the citizenship of a foreign
State, or is under any acknowledgment of allegiance or adherence to a foreign State;
o if he is so disqualified by or under any law made by Parliament.
Apart from these, article 101 describes conditions in which a member has to vacate his/her
seat. These are resignation, dual membership and absence from the house for a period more
than 60 days.

37. Which of the following statements are correct regarding Joint Session of the Houses of the
Parliament in India?
1. It is an enabling provision, empowering the President to take steps for resolving deadlock
between the two Houses.
2. It is not obligatory upon the President to summon the Houses to meet in a joint sitting
3. It is being notified by the President
4. It is frequently resorted to establish the supremacy of the Lok Sabha.
Select the correct answer using the code given below.
(a) 1 and 2 only (b) 1, 2 and 3 only
(c) 2 and 3 only (d) 3 and 4 only

Answer: (b)
Explanation
Joint Sitting is an enabling provision provided by the constitution for resolving a
disagreement between two houses of parliament.
The president may notify to the houses his intention to summon joint sitting in case of
disagreement between the two houses. Even though the Lok Sabha has an upper hand in
joint sitting by virtue of its numbers, it is a rare occurrence. President has called for a joint
sitting only thrice.

38. Which of the statements relating to the Deputy Speaker of the Lok Sabha is/are correct?
1. The office of the Deputy Speaker acquired a more prominent position after the
enforcement of the Constitution of India in 1950.
2. He/she is elected from amongst the members.
3. He/she holds office until he/she ceases to be a member of the House.
Select the correct answer using the code3 given below.
(a) 1 only (b) 1 and 2 only
(c) 1, 2 and 3 (d) 2 and 3 only

Answer: (c)
Explanation
Office of the deputy speaker acquired a more prominent position after the enforcement of the
constitution. He is elected amongst the members as soon as after its first sitting. As per
Article 94 he holds office until he ceases to be a member of the House/resigns/resolution for
removal by a majority of all then members of the house.

39. Which one among the following is a function of the Pro-Tem Speaker of the Lok Sabha?
(a) Conduct of the proceedings of the house in the absence of the Speaker
(b) To check if the election certificates of the members of the house are in the order
(c) Swear in the members of the House and hold the charge till a regular Speaker in the
members of the House is elected
(d) Give his assent to the bills passed by the House


Answer: (c)
Explanation
When the offices of both the Speaker and the Deputy Speaker fall vacant, the duties of the
Office of the Speaker are performed by pro-term speaker appointed by the president.
After the General Elections, the Speaker pro tem is appointed to administer
oath/affirmation to the newly elected members of Lok Sabha and conduct the election of
the Speaker. He vacates Office soon after the Speaker is chosen by the House. The Speaker
pro tem takes oath as a member of Lok Sabha before the President.

40. The principle of collective responsibility under parliamentary democracy implies that
1. A motion of no-confidence can be moved against the Council of Ministers as a whole as
well as against an individual minister
2. No person shall be nominated to the Cabinet except on the advice of the Prime Minister
3. No person shall be retained as a member of the Cabinet if the Prime Minister says that he
shall be dismissed
Select the correct answer using the code given below.
(a) 1 only (b) 2 only
(c) 3 only (d) 2 and 3 only

Answer: (d)
Explanation
Council of Ministers is collectively responsible to Lok Sabha and council of ministers can be
in power only as far as they enjoy the support in the Lok Sabha. No-confidence motion can be
moved against the council of ministers only.
All the ministers are appointed by the president on the advice of the Prime Minister. It is the
Prime Minister who allocates the portfolio to other ministers. The prime Minister may call for
the resignation of any minister at any time. In case the minister refuses, the prime minister
may advice the President to dismiss the minister.

41. Consider the following statements about the powers of the President of India:
1. The President can direct that any matter on which decision has been taken by a Minister
should be placed before the Council of Ministers.
2. The President can call information relating to proposals for legislation.
3. The President has the right to address and send messages to either House of the
Parliament.
4. All decisions of the Council of Ministers relating to the administration of the Union must
be communicated to the President.
Which of the statements given above are correct?
(a) 1, 2 and 3 only (b) 1 and 3 only
(c) 2 and 4 only (d) 1, 2, 3 and 4

Answer: (d)
Explanation
Article 78 clearly provides that
It shall be the duty of the Prime Minister
a. to communicate to the President all decisions of the Council of Ministers relating to the
administration of the affairs of the Union and proposals for legislation;
b. to furnish such information relating to the administration of the affairs of the Union and
proposals for legislation as the President may call for; and
c. if the President so requires, to submit for the consideration of the Council of Ministers any
matter on which a decision has been taken by a Minister but which has not been
considered by the Council.
Article 86 clearly delineates the right of the president with respect to address and send
messages to both houses.

42. Which of the following is/are part/parts of the procedure for the removal of a Judge of the
Supreme Court of India?
1. A motion signed by at least 100 members of Lok Sabha or 50 members of Rajya Sabha is
delivered to the Speaker or Chairman.
2.


3. The motion is investigated by a Committee of three Jurists constituted by the Speaker or
Chairman.
4. The Judge will be removed by the Speaker or Chairman if the Committee of three Jurists
recommends.
Select the correct answer using the code given below.
(a) 1, 2 and 3 (b) 1 and 2 only
(c) 2 and 3 only (d) 1 only

Answer: (b)
Explanation
Article 124(4) of the constitution and the Judges (Inquiry) Act 1968 together lay down an
elaborate procedure for the removal of the Supreme Court judge.
It involves following steps:
A motion for the removal of a judge, signed by at least 100 members of Lok Sabha or 50
members of Rajya Sabha is delivered to the presiding officer of the house.
The presiding officer constitute a committee of three jurists to investigate the motion.
If the committee recommends the removal of the judges, the report of the committee is
taken up for the discussion.
If the motion is passed in each house by a majority of the total membership the House
and by a majority of not less than two-thirds of members present and voting, the
address is presented to the President
Finally, president gives order for the removal of the judge.
Therefore, it is the President who removes the judge, not the speaker or chairman. The same
procedure is followed for high court judges as well.
43. Which one among the following disputes is not included in the original jurisdiction of the
Supreme Court of India?
(a) Between the Government of India and one or more States
(b) Between the Government of India and one or more citizens of India
(c) Between the Government of India and any State or States on one side and one or more
States on the other
(d) Between two or more States

Answer: (b)
Explanation
Original Jurisdiction means the power to hear a dispute in the first instance, not by way of
appeal. Article 131 of the constitution mentions about the original jurisdiction of the
Supreme Court. And it mainly confines to federal disputes. There are:
o between the Centre and the State or
o between the Centre and States on one side and one or more States on the other side or
o between the State themselves as issues relating to the distribution of powers.

44. On which of the following grounds can a Judge of the Supreme Court or a High Court be
removed?
1. Violation of the Constitution
2. Proved misbehaviour
3. Incapacity
Select the correct answer using the code given below.
(a) 1 only (b) 2 only
(c) 1, 2 and 3 (d) 2 and 3 only

Answer: (d)
Explanation
Article 124(4) mentions the grounds upon which removal may take place. These are proved
misbehaviour and incapacity of the judge.
Misbehavior can be wilful abuse of constitutional office, wilful misconduct in the office,
corruption, lack of integrity or any other offence involving moral turpitude by the judge.

45. Under the Constitution of India, which one of the following is not a fundamental duty?
(a) To abide by the Constitution and respect its ideals.


(b) To protect and improve natural environment
(c) to promote harmony and the spirit of common brotherhood
(d) To pay taxes

Answer: (d)
Explanation
Part IV-A consist of only one Article, Article 51-A imposes 11 fundamental duties for all Indian
citizens.
(a) to abide by the Constitution and respect its ideals and institutions, the National Flag and the
National Anthem.
(b) to cherish and follow the noble ideals of freedom struggle.
(c) to uphold and protect the sovereignty, unity and integrity of India.
(d) to defend the country and render national service when called upon to do so.
(e) to promote harmony and the spirit of common brotherhood amongst all the people of
India transcending religious, linguistic and regional or sectional diversities and to
renounce practices derogatory to the dignity of women.
(f) to value and preserve the rich heritage of the countrys composite culture.
(g) to protect and improve the natural environment (forests, lakes, rivers and wildlife) and to
have compassion for living creatures.
(h) to develop scientific temper, humanism and the spirit of inquiry and reform
(i) to safeguard public property and to abjure violence.
(j) to strive towards excellence in all sphere of individual and collective activity so that the
nation constantly rises to higher levels of endeavour and achievement.
(k) duty of parents or guardians to provide opportunities for education to their children or
ward between 6-14 years of age.
46. Which of the following are included in the Constitution of India as the Directive Principles of
State Policy?
1. Separation of Judiciary from Executive
2. Participation of workers in management of industry.
3. Equal justice and free legal aid.
Select the correct answer using the code given below.
(a) 1 and 2 only (b) 2 and 3 only
(c) 1 and 3 only (d) 1, 2 and 3

Answer: (d)
Explanation
Part IV containing Articles 36 to 51 of the Constitution deals with Directive Principles of State
Policy. It may be described as the active obligation of the State. They are designed to foster
social and economic democracy in the country. These principles obligate the State to take
positive action in certain directions in order to promote the welfare of the people. Article 37
itself says that the Directive Principles are fundamental in the governance of the country and
It shall be the duty of the State to apply these principles in the lawmaking.

47. Which of the following is/are included in the Directive Principles of State Policy?
1. Protection and improvement of the natural environment and safeguarding the forest and
wildlife
2. Organization of agriculture and animal husbandry on modern and scientific lines
Select the correct answer using the code given below.
(a) 1 only (b) 2 only
(c) Both 1 and 2 (d) Neither 1 nor 2

Answer: (c)
Explanation
Directive Principles are fundamental in the governance of the country. Article 48 A says that
State shall endeavor to protect and improve the natural environment and to safeguard the
forest and wildlife. It was inserted by the 42nd Amendment Act, 1976. Since State is the
custodian of the natural resources, it has the duty to maintain them not merely for the
benefit of public trust, but for the best interest of flora and fauna, wildlife and so on. Article


48 deals with oraganisation of agriculture and animal husbandry including steps for
preserving and improving the breeds, prohibition of cow slaughter etc.

48. Consider the following statements:


1. While the Fundamental Rights constitute limitations on State action, the Directive
Principles are in the nature of instruments of instruction to the government of the day to
do certain things and to achieve certain goals by their actions.
2. The Directive Principles, however, require to be implemented by legislations and so long
as there is no law carrying out the policy laid down in a Directive Principle, neither the
State nor an individual can violate any existing law or legal rights under the colour of
getting a Directive.
3. The Directive Principles are enforceable in the courts and create justiciable rights in
favour of the individual.
Which of the statements given above is/are correct?
(a) 1, 2 and 3 (b) 1 and 3 only
(c) 1 and 2 only (d) 2 only

Answer: (c)
Explanation
FR are negative obligations of the State and they are in the form of prohibiting the State from
performing certain acts whereas, DPSP are positive duties cast upon the State to attain the
ideal of social and economic justice. FR are enforceable in a court of law, but DPSP are non-
justiciable. i.e., cannot be enforced in a court of law. Although Directive Principles, as such,
were legally non-enforceable, nevertheless, while interpreting a statute, the courts shall have
recourse to the Directive Principles. The Courts therefore could interpret a statute so as to
implement Directive Principles instead of reducing them to mere theoretical ideas.
Fundamental Rights should be interpreted in the light of the Directive Principles and the
Directive Principles should be read into the Fundamental Rights. The political rights
(Fundamental Rights) cannot be realised without realisation of economic and social rights
(Directive Principles).

49. Consider the following with regards to 97th Constitutional Amendment Act
1. It inserted right to form Co-operative as a fundamental right.
2. It fixed minimum and maximum numbers of directors allowed for a Co-operative.
Which of the statements given above is/are correct?
(a) 1 only (b) 2 only
(c) Both 1 and 2 (d) Neither 1 nor 2
Answer: (a)
Explanation
The Parliament passed the Constitution (Ninety Seventh Amendment) Act, 2011 with an
objective to promote voluntary formation, autonomous functioning, democratic control and
professional management of Co-operative Societies. It inserted Co-operative Societies in
Article 19(1)(c); inserted Article 43B in Part IV and also inserted Part IXB in the Constitution
containing Articles 243ZH to 243ZT. Various restrictions have been imposed upon the State
Legislatures while enacting law relating to Co-operative Societies which was earlier unfettered
prior to the incorporation of Part IXB.
Article 243ZJ provides that maximum number of Directors of a Co-operative Society shall not
exceed twenty-one. But the Constitutional Amendment Act has not fixed any minimum
number of Directors for a Co-operative.

50. With reference to the Constitution of India, consider the following:


1. Fundamental Rights
2. Fundamental Duties
3. Directive Principles of State Policy
Which of the above provisions of the Constitution of India is/are fulfilled by the Swachh
Bharat Abhiyan launched by the Government of India?
(a) 1 only (b) 3 only
(c) 1 and 3 only (d) 1, 2 and 3


Answer (d)
Explanation
The Swachh Bharat Abhiyan is a massive movement that seeks to create a Clean India. It
ensures clean environment which is implied under the right to life, a fundamental right.
Similarly, Article 47, directs the state to improve public health and raise the standard of
living. It is related with the denouncing practices derogatory to the dignity of women and
improving the environment, which is a fundamental duty.

51. Which among the following states are considered under the Sixth Schedule of the
Constitution for the purpose of granting maximum autonomy to tribal areas?
1. Assam 2. Nagaland
3. Meghalaya 4. Mizoram
5. Arunachal Pradesh
Select the correct answer using the code given below.
(a) 1, 2 and 5 (b) 1, 3 and 4
(c) 2 and 5 (d) 1, 2, 3, 4 and 5

Answer (b)
Explanation
The Sixth Schedule under Article 244(2) of the Constitution relates to those areas in the
States of Assam, Meghalaya, Tripura and Mizoram which are declared as Tribal Areas.

52. Consider the following statements:


1. Chief Secretary heads the state administrative machinery whereas DGP heads the state
police force.
2. DGP can be removed from the post if he/she loses confidence of Chief Minister where as
Chief Secretary of the State enjoys security of tenure.
Which of the statements given above is/are correct?
(a) 1 only (b) 2 only
(c) Both 1 and 2 (d) Neither 1 nor 2

Answer (a)
Explanation
While DGP heads the State Police Force, Chief Secretary heads the state administrative
machinery. Both are senior civil servant officers in highest ranks. However, there is a slight
difference in their service conditions as clarified by the recent Supreme Court verdict on T.P
Senkumar case.
As per the judgment, DGP should have two years of fixed tenure. They cannot be transferred
or removed arbitrarily on the sole ground of lack of confidence in him by the Chief Minister.
On the other hand, Chief Secretary has pivotal role in overseeing the general administration.
Therefore, Chief Secretary must always enjoy the trust of Chief Minister for the larger interest
of general administration.

53. Indian President is elected from the electoral college consisting of


1. All members of the Lok Sabha
2. All elected members from Rajya Sabha
3. All elected members of Legislative Assemblies
4. All elected members of Legislative Councils
Select the correct answer using the codes below.
(a) 1, 2 and 4 only (b) 2 and 3 only
(c) 1, 3 and 4 only (d) 1, 2, 3 and 4

Answer (b)
Explanation
The President is elected by an indirect election, that is, by an electoral college, in accordance
with the system of proportional representation by means of single transferable vote by
secret ballot.
The electoral college shall consists of-
The elected members of both the Houses of Parliament;
The elected members of the Legislative Assemblies of the State


The elected members of the Legislative Assemblies of Union Territories of Delhi and
Pondicherry.

54. Which of the following Fundamental Rights is enjoyed by the Indian citizens alone?
(a) Right against exploitation
(b) Freedom of Speech and Expression
(c) Equality before law
(d) Right to Religion

Answer (b)
Explanation
Fundamental Rights granted to the citizens alone are
equality of opportunity in matters of public employment (Article 15)
protection from discrimination on any ground (Article 16)
freedom of speech, peaceful assembly, expression, association, movement, residence and
profession (Article 19)
cultural and educational rights (Article 29 and Article 30)

55. Consider the following with regard to the Overseas Citizenship:


1. Overseas Indian Citizens will be granted multiple-entry, multi-purpose life-long visa to
visit India.
2. They are exempted from registration with local police authority.
Which of the statements given above is/are correct?
(a) 1 only (b) 2 only
(c) Both 1 and 2 (d) Neither 1 nor 2
Answer (c)
Explanation
Citizenship Amendment Act 2003 provides for definition, registration, renunciation,
conferment of rights on overseas citizen and cancellation of overseas citizenship. An Overseas
Citizen of India is entitled to some benefits such as a multiple-entry, multi-purpose life-
long visa to visit India. He is exempted from registration with local police authority for any
length of stay in India. He enjoys parity with Non-Resident Indians in respect of economic,
financial and educational fields except in relation to the acquisition of agricultural or
plantation properties. However, he will not enjoy right to equality in public employment.
He will not enjoy right to vote and elect to Parliament and hold Constitutional offices.
Overseas citizenship can be cancelled on certain grounds which are same grounds on which
Indian citizenship can be terminated by deprivation i.e. citizenship was obtained by fraud,
showing disloyalty to the Constitution of India etc.
56. Which of the following actions may amount to corrupt practice in elections as per the
Representation of the People Act, 1951?
1. Offering dinner to vote for a particular candidate.
2. Hiring government vehicle for the campaigning.
3. Appealing to the national flag for furthering the prospect of a candidate.
4. Agent of the candidate offering free lift to the polling station.
5. Use of caste and language to canvass votes.
Select the correct answer using the code given below.
(a) 1 and 2 only (b) 3, 4 and 5 only
(c) 1, 2 and 5 only (d) 1, 2, 3, 4 and 5

Answer (d)
Explanation
As per Section 123 of RPA 1951, all of these constitute corrupt practices. Section 123(3)
was in news recently because of the landmark judgment by the Supreme Court that prohibit
the use of religion, race, caste or language as a ground for canvassing votes in an election.
Corrupt practices under section 123 are
giving bribery, undue rewards or gifts to voters
attempts to induce the voter to vote for any particular candidate or otherwise he will
become an object of divine displeasure.

promotion of hatred for campaigning
usage of religion, race, caste or language for canvassing votes
appealing to national symbols for the furtherance of prospects of the candidate
usage of government vehicles for campaigning
false and purposive allegation against opponent candidates personal character or
conduct
spending more money than allowed
Acceptance of any offer of liquor or other intoxicants or a dinner to vote for a particular
candidate or not to vote for him is bribery.
The provision of Corrupt Practice under section 123 (5) will cover conveyance of any elector,
to or from any polling station.

57. According to Article 370, most of the provisions in the Constitution of India can only be
extended to Jammu and Kashmir State through
(a) a legislation passed in the Jammu and Kashmir State Assembly
(b) a resolution passed in both houses of the Parliament
(c) a Presidential order issued in agreement with the state government
(d) a notification by the State Governor in agreement with the state government

Answer (c)
Explanation
Article 370 in the Constitution defines special provisions enjoyed by Jammu and Kashmir.
According to Article 370(d) Most of the provisions in the Constitution can be applied to the
state with such exceptions and modifications as specified by the President in consultation
with the state government or with the concurrence of the state government. The article also
grant power to Parliament to legislate on matters in Union and Concurrent list which
correspond to that specified in the Instrument of Accession. These are external affairs,
defence, communication and ancillary matters.

58. The authority to impose restriction on Inter-state trade lies with


1. State Legislature
2. President
3. Ministry of Commerce
4. Parliament
Select the correct answer using the code given below.
(a) 1 and 2 (b) 2 and 3
(c) 1 and 4 (d) 3 and 4

Answer (c)
Explanation
Article 301 of the Constitution extends right to freedom of inter-State trade and commerce
throughout the territory of India. The executive is incapable of imposing restrictions on the
freedom of trade and commerce without the sanction of legislature by law. Both the
Parliament and the State Legislature have been authorized to impose restrictions on the
freedom of inter-State trade provided it is in public interest. However, the restriction so
imposed by the Legislature is subject to Judicial Review and the restrictions shall be
reasonable.

59. Which of the following statements is correct with regard to Finance Commission?
(a) It assists the Parliament to scrutinize the CAG reports.
(b) Finance Commissions recommendations are binding on the Government.
(c) The Constitution mandates that the Chairman of the Commission must be a High Court
Judge or one qualified to be so.
(d) President is required to lay down the recommendations of the commission and action
taken on these before the Parliament.

Answer (d)
Explanation
Finance Commission recommend the procedures and principles guiding the distribution of
financial resources between the Union and the States. Its recommendations are not


binding on the government. Article 280 provides for the appointment by the President of a
Finance Commission consisting of a Chairman and four members every five years.
According to the Finance Commission (Miscellaneous Provisions) Act, 1951 passed to
supplement the Constitutional provisions, the chairman of the commission must be a person
having experience in public affairs. A High Court Judge or one qualified to be appointed as
such will serve as one of its members.

60. Consider the following statements:


The function(s) of the Comptroller and Auditor General of India is/are
1. to allow the withdrawal of the money out of the Consolidated Fund of India
2. to audit the expenditure from Consolidated Fund, Contingency Fund and Public Account
of India.
3. to prescribe the form in which accounts of Union and State shall be kept.
4. to compile and maintains accounts of Union Government.
Which of these statements given above is/are correct?
(a) 1 and 4 (b) 2 and 3
(c) 2 and 4 only (d) 1, 3 and 4

Answer (b)
Explanation
CAGs main function is to audit and report the expenditure from Consolidated Fund,
Contingency Fund and Public Account of India and that of states.
According to Article 150, the form in which the accounts of the Union and of the States are
to be maintained is to be determined by the President on the advice of the CAG.
CAG had earlier performed the function of compiling the accounts of the government. But the
Comptroller and Auditor-General (Duties, Powers and Conditions of Service) Act, 1971,
which, as amended in 1976, relieves CAG of its duty to compile the accounts of the Union.
However, Comptroller and Auditor General continues to compile the accounts of all States
except the State of Goa.

61. Consider the following statements in respect of financial emergency under Article 360 of the
Constitution of India:
1. A financial emergency is always declared along with National Emergency.
2. Once approved by the Parliament, the financial emergency shall remain in force for an
indefinite period till the President revokes it.
3. President may reduce salaries of Constitutional functionaries and civil servants while the
Financial Emergency is in force.
Which of the statements given above is/are correct?
(a) 1 and 2 only (b) 2 and 3 only
(c) 2 only (d) 1, 2 and 3

Answer (b)
Explanation
Financial Emergency is dealt under Article 360 while National emergency is declared under
Article 352. These are not declared concurrently.
Once approved the financial emergency shall remain in force for an indefinite period till the
President revokes the financial emergency.
When the financial emergency is enforced the distributions of financial resources between
Centre and State is suspended. The President may also reduce the salaries and allowances of
the Civil servants at the Union level and Constitutional functionaries of the Union, including
the judges of the Supreme Court and High Courts. . He may direct the Governor of State to
reserve all the money and the financial bills passed by the State Legislature for his
consideration.

62. Which of the following is not a ground for disqualification under Anti-Defection law?
(a) An elected member voluntarily resigns from his political party and joins any other political
party
(b) An elected member votes or abstains from voting contrary to party whip
(c) An independent member of legislature joins any political party


(d) A nominated member of a House joins a political party within six months after becoming a
member of the house

Answer (d)
Explanation
The grounds on which a member of the Parliament and State Legislature shall be disqualified
from being members of the House are:
If an elected member voluntarily gives up the membership of his political party and joins
any other political party.
If an elected member votes or abstains from voting in House contrary to any direction
issued by the political party to which he belongs.
If an independent member of a legislature joins political party after been elected to the
House.
If a nominated member of a House joins a political party six months after becoming a
member of the House

63. Consider the following statements:


The function(s) of Election Commission of India is/are
1. To cancel polls in the event of rigging or booth capturing.
2. To advise the Supreme Court on matters relating to the disqualifications of MPs.
3. To register and to grant them status of national or state parties.
4. To prepare and periodically revise electoral rolls.
Select the correct answer using the code given below.
(a) 1 only (b) 2 and 4 only
(c) 1, 3 and 4 only (d) 1, 2, 3 and 4

Answer (c)
Explanation
Election commission advises to President and not to Supreme Court on matters relating to
the disqualifications of MPs. Rest of the functions are performed by election commission.

64. Which of the following functions is/are performed by the Union Public Service Commission?
1. Pay and Service Conditions of Civil Servants.
2. Cadre Management of All India Services.
3. Conducting examinations for appointments in public banks.
Select the correct answer using the code given below.
(a) 1 only (b) 2 and 3
(c) 1 and 3 (d) None

Answer (d)
Explanation
Under Article 320 of the Constitution of India, the Commission is, inter-alia, required to be
consulted on all matters relating to recruitment to civil services and posts. The functions of
the Commission under Article 320 of the Constitution are:
Conduct examinations for appointment to the services of the Union.
Direct recruitment by selection through interviews.
Appointment of officers on promotion / deputation / absorption.
Framing and amendment of Recruitment Rules for various services and posts under the
Government.
Disciplinary cases relating to different Civil Services.
Advising the Government on any matter referred to the Commission by the President of
India.
Cadre Management, Pay and Service Condition of civil setvants, training etc. are functions
handled by Department of Personnel and Training.

65. An Attorney General appointed by the President of India must be a person qualified to be
appointed as a
(a) Chief Justice of high court
(b) Supreme Court Judge


(c) High Court Judge
(d) Senior most advocate in Supreme Court

Answer (b)
Explanation
According to Article 76 of the Constitution the President shall appoint a person who is
qualified to be appointed a Judge of the Supreme Court to be Attorney-General for India.
Article 124 (3) of the Constitution prescribes that for appointment as a judge of the Supreme
Court a person must he
a citizen of India,
has been a judge of any High Court for at least 5 years, or
has been an advocate in a High Court for 10 years or is in the opinion of the President a
distinguished jurist.
Therefore above mentioned qualifications are required by the Attorney General as well.

66. Which of the following are the expenditure charged on the Consolidated Fund of India?
1. Salary and Allowances of High Court Judges
2. Emoluments and allowance of the President
3. Pensions of High Court Judges
4. Debt charges of Government of India
5. Salary and pensions of Comptroller and Auditor General of India
Select the correct answer using the codes below.
(a) 1, 2 and 5 (b) 1, 3 and 4
(c) 2, 3, 4 and 5 (d) 1, 2, 3, 4 and 5

Answer (c)
Explanation
Following are the charged expenditures: Salary and Allowances of the President, Speaker /
Deputy Speaker of Lok Sabha, Chairman/ Deputy chairman of Rajya Sabha, Salaries and
Allowances of Supreme Court judges, Pensions of Supreme Court as well as High Court
Judges, Salaries and Allowances of CAG, Lok Pal and Debt charges of Government of India.
Salary of high court judges are charged from the consolidated fund of the state.
The above expenditures cannot be voted because these payments are deemed to be
guaranteed by the state. Although voting does not take place on such charges, discussion
can take place in any house of the Parliament. The demand for grant for these charges is
also made on recommendation of the President.

67. Consider the following statements with regard to 97th Constitutional Amendment Act:
1. It inserted right to form cooperative as a Fundamental Right.
2. State Governments power to regulate cooperatives became enhanced.
3. It fixed minimum and maximum numbers of directors allowed for a co-operative.
Which of the statements given above is/are correct?
(a) 1 only (b) 2 and 3 only
(c) 1 and 3 only (d) 1, 2 and 3

Answer (a)
Explanation
Various restrictions have been imposed upon the State Legislatures while enacting law
relating to Co-operative Societies which was earlier unfettered prior to the incorporation of
Part IXB.
Article 243ZJ provides a definite restriction which has been imposed upon the State
Legislatures regarding fixation of maximum number of Directors of a Co-operative Society
which shall not exceed twenty-one. The Constitution does not provide for minimum number
of directors.
As per the amendment the changes done to Constitution are:-
o In Part III of the Constitution, after words or unions the words Cooperative Societies
was added.
o In Part IV a new Article 43B was inserted, which says: The state shall endeavour to
promote voluntary formation, autonomous functioning, democratic control and
professional management of the co-operative societies.


o After Part IXA of the Constitution, a Part IXB was inserted to accommodate state vs centre
role

68. The elected members of State Legislative Assemblies can participate in


1. Electing President
2. Electing Vice-President
3. Electing members of State Legislative Council
4. Impeaching the President
5. Removing High Court judge
Select the correct answer using the code given below.
(a) 1, 2 and 5 (b) 2, 3 and 4
(c) 1 and 3 (d) 1, 4 and 5
Answer (c)
Explanation
Removing a high court/Supreme Court judge, impeaching the President and electing vice-
President are exclusive powers of Parliament. State legislative assembly members do not
take part in these.

69. Which among the following bodies embody the principle of cooperative federalism in its
working?
1. Monetary Policy Committee
2. GST Council
3. National Institution for Transforming India
Select the correct answer using the code given below.
(a) 1 and 2 only (b) 2 and 3 only
(c) 1 and 3 only (d) 1, 2 and 3

Answer (b)
Explanation
Cooperative federalism refers to participation of both Union and Federal units in the
governance of the country in a cooperative manner.
In the recently established GST Council, decisions are taken jointly by Union Government
and State Governments. NITI Aayog also enables participation and support to state
governments in the development and governance activities. It brings States together in
national interest, and thereby fosters Cooperative Federalism.
On the other hand, Monetary Policy Committee is a committee of the Central Bank in India
which is entrusted with the task of fixing the benchmark policy interest rate. It has
participation from central government and RBI and does not involve state governments.

70. Consider the following statements with regard to National Commission for Backward Classes?
1. It is a Constitutional body set up for the welfare of the Socially and Educationally
backward classes.
2. It recommends the inclusion or exclusion of communities from OBC Central list.
3. The body has the authority to prosecute government officers who fail to ensure mandated
reservation for OBCs.
Which of the statements given above is/are correct?
(a) 1 and 2 only (b) 2 and 3 only
(c) 2 only (d) 1, 2 and 3

Answer (c)
Explanation
National Commission for Backward Classes is set up through the enactment of National
Commission for Backward Classes Act, 1993 to determine and recommend both addition
and omission of different classes under OBC list. It is not a Constitutional body as of now.
The Commission consists of five Members, comprising of a Chairperson who is or has been a
judge of the Supreme Court or of a High Court; a social scientist; two persons, who have
special knowledge in matters relating to backward classes; and a Member-Secretary, who is
or has been an officer of the Central Government in the rank of a Secretary to the


Government of India. The Commission have all the powers of a civil court. And the advice of
the Commission shall ordinarily be binding upon the Central Government.
The commission was on news because Ministry of Social Justice has put forward a new
Constitutional Amendment bill to replace existing commission with more powers and to grant
Constitutional status.

71. Which of the followings is not a department under the Ministry of Finance?
(a) Department of Disinvestment
(b) Department of Expenditure
(c) Department of Financial Service
(d) Department of Investment and Public Asset Management

Answer (a)
Explanation
Various departments in the Finance Ministry are
Economic Affairs
Expenditure
Revenue
Financial Services
Department of Investment and Public Asset Management

72. National Human Rights Commission can inquire into a human right violation case
1. if a petition is filed before it
2. when court directs to conduct investigation
3. by way of suo moto
Select the correct answer using the code given below.
(a) 1 and 2 only (b) 2 and 3 only
(c) 1 and 3 only (d) 1, 2 and 3

Answer (d)
Explanation
NHRC can take up a case on its own initiative (suo moto) or on a petition presented to it by a
victim or any person on his behalf. It can also take up a case when courts directs.

73. Consider the following with regard to Central Administrative Tribunals:


1. Administrative tribunals are Constitutional bodies to provide for speedy disposal of
disputes relating to service matters.
2. They have the power to adapt their own rules of procedure.
Which of the statements given above is/are correct?
(a) 1 only (b) 2 only
(c) Both 1 and 2 (d) Neither 1 nor 2

Answer (b)
Explanation
Article 323A empowers the Parliament by law for the establishment of administrative
tribunals and Article 323B for the establishment of tribunals such as income tax, land
dispute tribunal etc. Since they are established through Law rather than a Constitutional
Amendment Act, they are statutory bodies. It is also confirmed by Supreme Court saying
that the administrative tribunals are statutory bodies.
The Administrative Tribunals are not bound by the technical rules of Civil Procedure Code
and Evidence Act. They have the power to adapt their own rules of procedure. They enjoy
some of the powers of civil court in certain matters and proceedings are constrained to be
judicial in nature.

74. Consider the following with regard to Cabinet Secretary:


1. His main function is to coordinate between different ministries.
2. Apex Committee on Digital India Programme is headed by Cabinet Secretary.
Which of the statements given above is/are correct?
(a) 1 only (b) 2 only
(c) Both 1 and 2 (d) Neither 1 nor 2


Answer (c)
Explanation
Cabinet Secretarys main function is to coordinate between different ministries and
departments for the smooth transaction of functions.
An Apex Committee headed by Cabinet Secretary is also responsible for overseeing the Digital
India programme and providing policy and strategic directions for its implementation and
resolving inter-Ministerial issues.

75. Consider the following pairs:


Department Ministry
1. Department of Space : Ministry of Science and Technology

2. Department of : Ministry of Law and Justice


Justice

3. Department of States : Ministry of Home Affairs

Which of the above pairs is/are correctly matched?


(a) 1 and 2 only (b) 2 and 3 only
(c) 1, 2 and 3 (d) None

Answer (b)
Explanation
Department of Space is an independent Ministry. Department of Atomic research is also an
Independent Ministry.
Department of Justice is a department under Ministry of Law and Justice. Department of
States is under the Ministry of Home Affairs.

76. Which of the following is/are the function/functions of Ministry of Parliamentary Affairs?
1. Forming Consultative Committees of Members of Parliament for various Ministries.
2. Pursues with the other Ministries for implementation of assurances give in the
Parliament.
3. Drafting the Legislations on behalf of different Ministries.
Select the correct answer using the code given below.
(a) 1 only (b) 2 and 3 only
(c) 1 and 2 only (d) 1, 2 and 3

Answer (c)
Explanation
Drafting legislation is mainly the responsibility of Ministry of Law and Justice. Ministry of
Parliamentary Affairs keeps a watch over the progress of Bills from the stage of approval
by the Cabinet till the Bill is passed by both Houses of the Parliament. In order to ensure
smooth passage of Bills, in Parliament, officials of the Ministry have to be in constant touch
with the officials of Ministries/Departments sponsoring the Bills and the Ministry of Law,
Justice and Company Affairs which drafts the Bills.

77. Appointment of Parliamentary Secretaries are discouraged and termed it as an


unconstitutional practice by various courts, because
(a) it will lead to overlapping functions of different Ministers hindering the administration.
(b) there is ample number of Civil Servants for assistance.
(c) they perform primarily the functions of civil servants.
(d) legislators would be obliged to the executives.

Answer (d)
Explanation
Various States have been appointing Parliamentary Secretaries to assist cabinet miniters.
They are usually have the same status as Minister of States. It is argued that these
Parliamentary Secretaries hold the Office of Profit and must be disqualified. However, it is yet
to emerge a clear picture on the exact nature of the position.
Various Courts, however, seen the practice as unconstitutional because it will contravene the
Constitutional provision on maximum number of Ministers. Also since they work as part of


executive, these legislators may obliged to executive. Therefore, the practice need to be
discouraged.

78. Consider the following with respect to Prime Miniters Office:


1. The Prime Ministers Office is headed by Cabinet Secretary.
2. The main function of Prime Ministers Office is inter-Ministerial coordination and framing
rules of business.
Which of the statements given above is/are correct?
(a) 1 only (b) 2 only
(c) Both 1 and 2 (d) Neither 1 nor 2

Answer (d)
Explanation
Prime Ministers Office [PMO] assists the Prime Minister in the efficient discharge of his role,
functions and responsibilities. It is headed by a Principal Secretary who has always been a
senior civil servant or a retired bureaucrat.
Inter-Ministerial coordination and framing rules of business are functions of Cabinet
Secretariat, not the responsibility of PMO. But PMO may engage in any inter-Ministerial
conflict so as to resolve it.

79. Which of the following amendments to the Indian Constitution is incorporated through 42nd
Constitutional Amendment Act, 1976, also known as mini-Constitution?
1. Added the phrases socialist and secular to the Preamble of Indian Constitution.
2. Provided restrictions in proclamation of National Emergency.
3. Added Fundamental Duties into the Constitution.
4. Restricted the discretionary power of the President.
Select the correct answer using the code given below.
(a) 1, 2 and 3 only (b) 1, 3 and 4 only
(c) 2 and 4 only (d) 1, 2, 3 and 4

Answer (b)
Explanation
Some of the Important changes brought by 42nd Constitutional Amendment Act, 1976
The preamble has been amended to substitute the words SOVEREIGN DEMOCRATIC
REPUBLIC, with the words SOVEREIGN SOCIALIST SECULAR DEMOCRATIC
REPUBLIC and the words unity of the Nation was substituted with unity and integrity
of the Nation.
New directives was added by new articles 39A, 43A, 48A which, respectively, provide for
equal justice and free legal aid to economically backward classes, participation of workers
in the management of industries, and protection and improvement of environment and
safeguarding of forests and wildlife.
New Part IVA containing article 51A was added to provide lists of fundamental duties of
citizens.
Article 74(1) was amended to make the President to act in accordance with the advice of
the Council of Ministers.
It curtailed the power of the Supreme Court and High Court with regard to the issue of
writs and judicial review.
It transferred subjects like forests, education, weights and measures except
establishments of standards, protection of wild animals and birds from the State List to
the Concurrent List. New entry 20A was added in Concurrent List which is Population
control and family planning.

80. Which of the following are Fundamental Duties as listed in the Constitution?
1. To safeguard public property and to abjure violence.
2. To promote international peace and universal brotherhood.
3. To value and preserve the rich heritage of our composite culture.
4. To develop a sense of respect for elders and to help disabled.
Select the correct answer using the code given below.
(a) 1 and 2 (b) 1 and 3


(c) 2, 3 and 4 (d) 1, 2, 3 and 4

Answer (b)
Explanation
List of Fundamental Duties: Art. 51A, Part IVA of the Indian Constitution, specifies the list of
fundamental duties of the citizens. It says it shall be the duty of every citizen of India:
to abide by the Constitution and respect its ideal and institutions;
to cherish and follow the noble ideals which inspired our national struggle for freedom;
to uphold and protect the sovereignty, unity and integrity of India;
to defend the country and render national service when called upon to do so;
to promote harmony and the spirit of common brotherhood amongst all the people of
India transcending religious, linguistic and regional diversities, to renounce practices
derogatory to the dignity of women;
to value and preserve the rich heritage of our composite culture;
to protect and improve the natural environment including forests, lakes, rivers, and wild-
life and to have compassion for living creatures;
to develop the scientific temper, humanism and the spirit of inquiry and reform;
to safeguard public property and to abjure violence;
to strive towards excellence in all spheres of individual and collective activity, so that the
nation constantly rises to higher levels of endeavor and achievement.
who is a parent or guardian, to provide opportunities for education to his child, or as the
case may be, ward between the age of six and fourteen years.

81. Consider the following statements:


1. The minimum age required to be a member in the State Legislative Council is 25.
2. State Legislative Councils cannot be dissolved.
Which of the statements given above is/are correct?
(a) 1 only (b) 2 only
(c) Both 1 and 2 (d) Neither 1 nor 2
Answer (b)
Explanation
MLC must be at least 30 years old. It is a permanent body and cannot be dissolved; each
MLC serves for a six-year term.

82. Consider the following with regard to Office of Profit:


1. It is not mentioned in the Constitution.
2. Disqualification of MPs on holding the post of Office of Profit is decided by the Speaker of
the House.
Which of the statements given above is/are correct?
(a) 1 only (b) 2 only
(c) Both 1 and 2 (d) None of the above

Answer (d)
Explanation
Office of Profit was frequently in news because of the controversy surrounding appointment
of Parliamentary Secretaries in Delhi. The term office of profit has mentioned in the
constitution but not defined in the Constitution. Article Articles 102 (1) and 191 (1)
which give effect to the concept of office of profit -- prescribe restrictions at the central and
state level on lawmakers accepting government positions. Any violation attracts
disqualification of MPs or MLAs.
If any question arises as to whether a Member of Parliament has become subject to any of the
disqualifications laid down in the Constitution, including the one whether he is holding an
office of profit or not, the question is referred for the decision of the President and his
decision is final. However, before giving any decision on any such question, the President is
required to obtain the opinion of the Election Commission and shall act according to such
opinions. It is important to note that in this matter the President does not act on the advice
of his Council of Ministers.


83. Consider the following with regard to Inter-State Council:
1. It is a Constitutional body set up to facilitate coordination of policies between the Union
and the State Governments.
2. All Union Cabinet Ministers and State Chief Ministers are members of the Council.
Which of the statements given above is/are correct?
(a) 1 only (b) 2 only
(c) Both 1 and 2 (d) None of the above

Answer (a)
Explanation
Article 263 of the Constitution of India provides for the establishment of an Inter -State
Council. Council charged with the duty of
Enquiring into and advising upon disputes which may have arised between States;
investigating and discussing subjects in which some or all of the States, or the Union
and one or more of the States, have a common interest
making recommendations upon any such subject
The Council is a recommendatory body.
The Council shall consist of
Prime Minister - Chairman
Chief Ministers of all States - Member
Chief Ministers of Union Territories having a Legislative Assembly and Administrators
of UTs not having a Legislative Assembly - Member
Six Ministers of Cabinet rank in the Union Council of Ministers to be nominated
by the Prime Minister - Member

84. Consider the following with regard to Armed Forces (Special Powers) Act (AFSPA):
1. The Act can only be imposed when a martial law under Article 34 is in force in an area or
a State.
2. Governor of the State and Central Government are empowered to declare any part or full
of any state as disturbed area.
Which of the statements given above is/are correct?
(a) 1 only (b) 2 only
(c) Both 1 and 2 (d) None of the above

Answer (b)
Explanation
AFSPA can be imposed in any place if its found to be a disturbed area regardless of whether
martial law is in force or not.
Governor of the State and Central Government are empowered to declare any part or full of
any state as disturbed area if it is in their opinion that it is necessary to prevent terrorist
activity or any such activity that might disrupt the sovereignty of India or cause insult to
the national flag, anthem or Indias Constitution. Section 4 of the act gives special powers to
army officers in disturbed area to shoot (even if it kills) any individual who violates law / or is
suspected violate law (this includes assembly of five or more people, carrying of weapons) etc.
The only condition is that the officer has to give warning before opening fire. Arrest anybody
without a warrant, and carry out searches without consent. Once a person is taken into
custody, he / she has to be handed over to the nearest police station as soon as possible.
Prosecution of the officer on duty needs prior permission of the Central Government.

85. Which of the following is/are the conditions suggested by the Election Commission of India to
recognize a party as a National Party?
1. It has to win a minimum of two per cent of the seats in the Lok Sabha from at least three
different States.
2. The party is recognised as a 'state level party' in four or more States.
3. In General Elections, the party must manage to win four per cent of the votes and win at
least six Lok Sabha seats.
Which of the statements given above is/are correct?
(a) 1 only (b) 2 and 3 only
(c) 1 and 2 only (d) 1, 2 and 3


Answer (c)
Explanation
Third statement must have been the other way around. The correct statement would be In
General Elections, the party must manage to win six per cent of the votes and win at least
four Lok Sabha seats.
A party has to live up to at least one of the following qualifications to be recognised as a
national party:
(a) It has to win a minimum of two percent of the seats in the Lok Sabha from at least
three different States.
(b) In General Elections, the party must manage to win six percent of the votes and win at
least four Lok Sabha seats as well.
(c) The party is recognised as a 'state level party' in four or more States.

86. Article 142 of the Indian Constitution provides discretionary powers to the Supreme Court to
ensure complete justice. In which of the following incidents, the Supreme Court has invoked
this Article while pronouncing the judgments?
1. Awarding compensation for Bhopal Gas tragedy victims.
2. Invalidating National Judicial Appointment Commission Act.
3. The ban on the sale of alcohol along highways.
4. Cancellation of discriminatory coal block allocation.
Which of the statements given above is/are correct?
(a)1, 2 and 3 only (b) 1, 3 and 4 only
(c) 2 and 4 only (d) 1, 2, 3 and 4

Answer (b)
Explanation
Article 142 provides that the Supreme Court in the exercise of its jurisdiction may pass
such decree or make such order as is necessary for doing complete justice in any cause or
matter pending before it This provision as a potent tool in the hands of the Supreme
Court to bring about changes in significant policy issues to affect the public at large. Earlier,
the Apex court has invoked this Article to bring complete justice to various deprived
sections of society or to protect the environment such as awarding compensation for
Bhopal Gas Tragedy Victims.
However, the unrestrained use of the Article has been criticized because it was seen as
Judicial overreach. For instance, the Supreme Court has invoked the same Article recently to
ban the sale of alcohol in highways and to cancel the coal block allocation. These were
essentially policy choices in the domain of executive action.
Supreme Court has struck down the NJAC Act because it affects the independence of
Judiciary, part of Basic Structure of the Constitution.

87. Consider the following statements with regard to the office of Parliamentary Secretary:
1. They are appointed by the Speaker of the Legislative Assembly to organize the smooth
functioning of the Assembly.
2. Parliamentary Secretaries will have access to all official files and documents.
Which of the statements given above is/are correct?
(a) 1 only (b) 2 only
(c) Both 1 and 2 (d) Neither 1 nor 2

Answer (b)
Explanation
Parliamentary secretaries are ruling party members appointed to assist ministers. They are
regarded for all practical purposes as deputy ministers having access to all official files
and documents. State Governments have been appointing parliamentary secretaries from
among MLAs, especially those who could not be included in the Council of Ministers as
junior ministers because of Article 164 (1A) which limits the total number of ministers to
15% of the strength of the assembly. In India, Chief Ministers appoint parliamentary
secretaries and administer oath of office. In general, the number of parliamentary secretaries
corresponds to the number of cabinet ministers.


88. Consider the following statements with regard to Article 370:
1. The power of the Parliament to make laws for Jammu and Kashmir is limited to foreign
affairs and defence only.
2. Article 370 cannot be repealed since it forms the basic structure of the constitution.
Which of the statements given above is/are correct?
(a) 1 only (b) 2 only
(c) Both 1 and 2 (d) Neither 1 nor 2

Answer (d)
Explanation
Article 370 is a temporary provision granting special autonomous status to Jammu and
Kashmir. Article 370 specifies that except for Defence, Foreign Affairs, and
Communications the Indian Parliament needs the State Governments concurrence for
applying all other laws. There is no State list for the State of Jammu and Kashmir. At the
same time, while in relation to the other States, the residuary power of legislation belongs to
Parliament, in the case of Jammu and Kashmir, the residuary powers belong to the
Legislature of the State except some matters that affect the integrity of the nation. The As per
the 3rd clause in the Article, Article 370 could be abrogated or amended by a Presidential
Order only upon the recommendation of the State's Constituent Assembly.

89. Memorandum of Procedure, often heard in the context of Indian Judiciary, is


1. the mechanism to clear the backlog of pending cases
2. the procedure to be followed in the appointment of judges
3. the rules followed in transfers of Judges
4. the procedure followed in filing Public Interest Litigation

Answer (b)
Explanation
Supreme Court had stuck down National Judicial Appointment Commission Act which
was aimed to give an important role for executive branch in Judicial appointment. At the
same time apex court has also agreed to revamp the present collegium system. They invited
the government to form consensus in forming Memorandum of Procedure in the
appointment of judges. However, serious disagreements between Judiciary and executive
prevails about whether the executive can have power to reject recommendations by the
court on the ground of national security.

90. Finance Commission is set up in every five years mainly to


1. examine whether the state and Central government fulfills the budgetary promises.
2. make recommendations to the central government for fiscal discipline.
3. decide on principles and procedures guiding the allocation of funds to States.
4. suggest measures to prevent money laundering.

Answer (c)
Explanation
Finance Commission is set up mainly to investigate and study about the financial
relation between the centre and the states.
It is the duty of the Finance Commission to make recommendations to the President as to:
the distribution of net proceeds of taxes between the Union and the States
the principles which should govern the grants-in-aid
the measures needed to augment the Consolidated Fund of a State to supplement the
resources of local self government.
any other matter referred to the Commission by the President in the interests of sound
finance.

91. Consider the following with regard to Union Public Service Commission:
1. UPSC chairman is selected by a Committee consisting of Prime Minister, Opposition
Leader in the Lok Sabha and the Lok Sabha Speaker.
2. UPSC Chairman is removed from the office in the same manner as a judge of Supreme
Court.


Which of the statements given above is/are correct?
(a) 1 only (b) 2 only
(c) Both 1 and 2 (d) Neither 1 nor 2

Answer (d)
Explanation
UPSC Chairman is selected among senior civil servants usually by the Government. There is
no such committee is constituted for the selection of UPSC Chairman. Such section
committees are formed for the appointments in statutory bodies like Chief Information
Commission, NHRC, CVC etc.
UPSC Chairman is removed by the President of India on grounds suggested in Constitution
(Article 317) and Parliament has no role to play in it like that of removal of Supreme Court
Judge.
A member or Chairman of UPSC can be removed by the President of India on any of the
following grounds. If the member or Chairman:
is adjudged an insolvent (unable to pay ones debt)
engages in paid employment during his tenure outside the duties of his office
is infirm of body or mind
participates in any office of profit
A member or Chairman of UPSC can be removed on ground of misbehavior, which is
proved in an inquiry conducted by the Supreme Court after a reference is made by the
President to the Supreme Court to conduct such an inquiry. The report by the Supreme
Court is binding on the President.
The Chairman or UPSC member can also be placed under suspension by the President until
report of Supreme Court into inquiry of alleged misbehavior is pending.

92. Which of the following are initiatives by NITI Aayog?


1. Atal Innovation Mission
2. Sankalp
3. DigiDhan Mela
4. Grand Innovation Challenge
Select the correct answer using the code given below.
(a) 1, 2 and 4 only (b) 2 and 3 only
(c) 1, 3 and 4 only (d) 1, 2, 3 and 4

Answer (c)
Explanation
Atal Innovation Mission and Self Employment and Talent Utilization are a scheme and an
approach respectively by the NITI Aayog to promote entrepreneurship.
Sankalp is a scheme launched by Ministry of Personnel, Public Grievances & Pensions aims
at channelizing skill, experience and time available with retired government servants into
meaningful, voluntary contribution to society.
DigiDhan Mela is a prizing program initiated by Niti Aayog to incentivize digital payments
Grand Innovation challenge by NITI Aayog seek citizens inputs on the key developmental
challenges facing India.

93. Performance Management Division which monitors the working efficiency of different
Ministries is under
1. The Ministry of Personnel, Public Grievances and Pensions
2. Cabinet Secretariat
3. Prime Ministers Office
4. NITI Aayog

Answer (b)
Explanation
Performance Management Division (PMD) in the Cabinet Secretariat is headed by a
Secretary to Government of India, and is responsible for this activity through the
mechanism called Results-Framework Documents (RFD).


At the beginning of each financial year, each department prepares a RFD consisting of the
priorities set out by the Minister concerned. At the end of the year, all
Ministries/Departments are required to review and prepare a report listing their respective
achievements against the agreed targets in the prescribed format.

94. Which among the following statement is not correct with respect to the parliamentary system
and the presidential system?
1. Governments in a Parliamentary system will be more stable.
2. A Parliamentary system allows better representation.
3. Presidential System is based on strict separation of power.
4. The Conflict between government and Parliament is easily resolved in a parliamentary
system.

Answer (a)
Explanation
In parliamentary form, the executive is formed from the legislature. For instance, in India,
Council of Ministers are selected from members of parliament. But in US which follows
presidential system has strict separation of power between executive and legislative and the
executive does not require the support from the legislature. Hence, it will be more stable
compared to parliamentary form. In parliamentary system, passing the non-confidence
motion is enough to overthrow the government.
Since the government is formed from the parliament which itself composed of representatives
from different regions and communities, Parliamentary system allows better representation in
government.

95. Which of the following are Classical Languages recognized by Government of India?
1. Sanskrit
2. Bangla
3. Odiya
4. Malayalam
Select the correct answer using the code given below.
(a) 1, 2 and 3 only (b) 1, 3 and 4 only
(c) 2 and 4 only (d) 1, 2, 3 and 4

Answer (b)
Explanation
Six languages are included in the list of Classical Languages:
Tamil
Sanskrit
Telugu
Kannada
Malayalam
Odiya
The government of India currently follows the following criteria to determine the eligibility of
language to be considered for classification as classical language:
High antiquity of its early texts/ recorded history over a period of 1500-2000 years.
A body of ancient literature/ texts, which is considered a valuable heritage by
generations of speakers.
The literary tradition be original and not borrowed from another speech community.
The classical language and literature being distinct from modern, there may also be a
discontinuity between the classical language and its later forms or its offshoots.

96. Which one of the following pairs of Schedules in the Constitution of India and its content is
not correctly matched?
Schedule Content
1. Tenth Schedule : Anti-defection
2. Eleventh Schedule : Allocation of seats in the Council of State
3. Seventh Schedule : Federal Structure
4. Eighth Schedule : Languages


Answer (b)
Explanation
Followings are the schedules in Constitution of India:
First Schedule List of States & Union Territories
Second Salary of President, Governors, Chief Judges, Judges of High
Schedule Court and Supreme court, Comptroller and Auditor General
Third Schedule Forms of Oaths and affirmations
Fourth
Allocation of seats for each state of India in Rajya Sabha
Schedule
Fifth Schedule Administration and control of scheduled areas and tribes
Provisions for administration of Tribal Area in Assam,
Sixth Schedule
Meghalaya, Tripura & Mizoram
Seventh
Allocation of powers and functions between Union & States.
Schedule
Eighth
List of 22 languages of India recognized by Constitution
Schedule
Ninth Schedule Acts that are protected from Judicial Review
Tenth
Anti Defection Law
Schedule
Eleventh
Panchayati Raj.
Schedule
Twelfth
Municipalities.
Schedule

97. Which one of the following statements with regard to the administration of Union Territories
is not correct?
(a) Puducherry and Delhi are administered through Lieutenant Governor.
(b) The Governor of Punjab functions as the Administrator of Chandigarh.
(c) The President appoints Advisory Committee for all Union Territories.
(d) President appoints Ministers of Puducherry and Delhi.

Answer (c)
Explanation
According to Article 239, The Union Territories are administered by the President through
an administrator, who is appointed by him with a suitable designation. This designation is
called either Lieutenant Governor or Chief Commissioner or Administrator. The two UTs
with Legislatures and the UT of Andaman and Nicobar Islands have Lt. Governors as
Administrators. The Governor of Punjab functions as the Administrator of the UT of
Chandigarh and a senior IAS officer functions as Advisor to the Administrator. The
remaining 3 UTs viz. Daman and Diu, Dadra and Nagar Haveli and Lakshadweep are
administered through IAS officers who are appointed as Administrators.
The President appoints Advisory Committee for each Union Territory, where there is no
representative government. The President appoints eminent citizens of Union Territory as
members of Advisory Committee. The President consults the Advisory Committee in matters
of governance. In Delhi and Puducherry, the President, not Lt. Governor, appoints Chief
Ministers and other Ministers.

98. Consider the following statements with regard to reservations in Urban Local Government
bodies:
1. Not less than one-third of the total number of seats in Municipalities will be reserved for
women.
2. States can prescribe the manner of reservation to the offices of the Chairpersons of
Municipalities.
Which of the statements given above is/are correct?


(a) 1 only (b) 2 only
(c) Both 1 and 2 (d) Neither 1 nor 2

Answer (c)
Explanation
Part IX A of the Indian Constitution contains detailed provisions for the administration of
Urban Local Bodies. Some of the provisions are similar to those contained in Part IX, Eg.
Reservation of Seats, Finance Commission, Election Commission etc.
This part gives birth to two type of bodies:
Institutions of self-government [Art. 243Q] Nagar Panchayat, Municipal Council
and Municipal Corporation.
Institutions of planning [Art. 243ZX and 243ZE] - Committee for Metropolitan
planning and Committee for district planning
According to Article 243 T, not less than one-third (including the number of seats reserved
for women belonging to the Scheduled Castes and the Scheduled Tribes) of the total number
of seats to be filled by direct election in every Municipality shall be reserved for women. It will
be allotted by rotation to different constituencies in a Municipality. It also provides that the
offices of Chairpersons in the Municipalities shall be reserved for the Scheduled Castes,
the Scheduled Tribes and women in such manner as the Legislature of a State may, by
law, provide.
However, please note that all the states have not implemented the womens reservation in
Municipalities. Nagaland did not reserve seats for women because Tribal groups in
Nagaland alleges that the reservation for women is against their customary practice. Naga
legislature is empowered to decide on the matter under Article 371A.

99. Which of the following special provisions is/are made in the Constitution for Hyderabad-
Karnataka region?
1. Establishment of a separate Development Board for the region.
2. Reservation in education and Government jobs for local people.
3. Special protection for the Scheduled Tribes in the region.
Select the correct answer using the code given below.
(a) 1 only (b) 1 and 2 only
(c) 2 and 3 only (d) 1, 2 and 3

Answer (b)
Explanation
98th Constitutional Amendment Act inserted Article 371 J which empower the Governor
of Karnataka to take steps to develop the Hyderabad-Karnataka Region. It provides for
establishing a separate development board and equitable allocation of funds for
development of the region. Apart from this, it will provide quota in public employment
through local cadres and reservation in education and vocational training institutions
for those who belong to the region by birth or by domicile. Special Status has been granted
considering the backwardness of the region rather than for the protection of tribal
population.

100. Consider the following statements with regard to Cabinet Secretary and Chief Secretary:
1. While Chief Secretaries are allotted responsibilities in departments or ministries, Cabinet
Secretary has no ministerial or departmental responsibilities.
2. Both are heads of their respective Civil Services Boards.
Which of the statements given above is/are correct?
(a) 1 only (b) 2 only
(c) Both 1 and 2 (d) Neither 1 nor 2

Answer (c)
Explanation
The similarities between Chief Secretary and Cabinet Secretary are:
1. Both are chief coordinators of their respective administrations.
2. Both are chief advisors to their respective Chief Executives.
3. Both are heads of their respective civil services boards.
4. Both are administrative heads to their respective secretariats.


5. Both supervise the implementation of the decisions of the respective cabinets.
The differences between the Chief Secretary and Cabinet Secretary are:
All those functions which are not specifically allotted to any other Secretary or
Additional Chief Secretary, fall within Chief Secretarys direct functional jurisdiction. But
at the Central level, Principal Secretary to the Prime Minister has the Residual
powers.
The Cabinet Secretary is primus inter pares (first among equals) among the Secretaries,
but the Chief Secretary is the head of all Secretaries (including Additional Chief
Secretaries and Principal Secretaries).
The Cabinet Secretary has no ministerial or departmental responsibilities, while a
Chief Secretary may be the administrative head of a few departments like Planning,
Personnel etc.
As per the emergent conventions, the Cabinet Secretary has a fixed tenure of minimum
two years, but such stipulation is not applicable in the case of the Chief Secretary.
The powers and functions of the Chief Secretary are wider and more intense than those
of the Cabinet Secretary.

You might also like